feat(analisi): aggiunge gli appunti del 30/03/2023

main
parent 49e61463e5
commit 10ff878080

@ -0,0 +1,195 @@
\documentclass[11pt]{article}
\usepackage{personal_commands}
\usepackage[italian]{babel}
\title{\textbf{Note del corso di Analisi Matematica 1}}
\author{Gabriel Antonio Videtta}
\date{30 marzo 2023}
\begin{document}
\maketitle
\begin{center}
\Large \textbf{Esercitazioni: applicazione dei teoremi sulla continuità}
\end{center}
\begin{exercise}
Siano $I = [a, b] \in \RRbar$, $f : I \to \RRbar$ continua strettamente
crescente. Allora l'eq.~$f(x)=y$ ha una sola soluzione se
$f(a) \leq y \leq f(b)$ e nessuna soluzione se $y < f(a)$ o
se $y > f(b)$.
\end{exercise}
\begin{solution}
Poiché $f$ è strettamente crescente, $f$ è iniettiva. Allora,
se $y$ è tale che $f(a) \leq y \leq f(b)$, per il teorema
dei valori intermedi, $\exists x \mid f(x) = y$; e tale $x$
è unica dal momento che $f$ è iniettiva. In particolare, poiché $f$ è crescente,
$f(a)$ e $f(b)$ sono rispettivamente $\inf f(I)$ e $\sup f(I)$, e
quindi sono anche
$\min f(I)$ e $\max f(I)$, da cui, se $y < f(a)$ o $y > f(b)$,
$y=f(x)$ non ammette soluzione.
\end{solution}
\begin{exercise}
Si consideri l'eq.~$xe^x = 4$ (*).
\begin{enumerate}[(a)]
\item Mostrare che (*) ammette un'unica soluzione $\xbar \in \RRbar$, e trovare $x_0$, $x_1$ tali che $x_0 < \xbar < x_1$.
\item Calcolare $\xbar$ con errore minore a $10^{-2}$.
\end{enumerate}
\end{exercise}
\begin{solution}
Si studia la funzione $f(x) = xe^x - 4$. $f$ è continua, e vale che
$f(0) = -4$ e che $f(2) = 2e^2 - 4 \geq 4$. Quindi, per il teorema
degli zeri su $[0, 2]$, $f$ ammette uno zero $\xbar$ in $(0, 2)$. \\
Si studia adesso la derivata $f'(x) = e^x + xe^x = (1+x)e^x$.
$f'(x) > 0 \iff x > -1$, ossia $f$ è crescente per $x > -1$.
Al contrario, $f$ decresce per $x < -1$; poiché allora
$\lim_{x \to -\infty} f(x) = -4$, $\sup f((-\infty), -1)) = -4$,
e quindi $f$ non ha zeri per $x < -1$, tantomeno per $x = -1$
(infatti $f(-1) = -1 e^{-1} - 4 \neq 0$). \\
Poiché per $x > -1$ $f$ è allora strettamente crescente,
$f$ può ammette un solo zero, ossia quello trovato all'inizio
della soluzione.
Per ricavare $\xbar$ con errore minore a $10^{-2}$, si applica
il metodo di bisezione per $7$ volte (infatti $\eps(n) = \frac{1}{2^n}$
per ogni passaggio $n$-esimo dell'algoritmo, e $\eps(7) \approx 0.0079 < 0.01$), ricavando $\xbar = 1.2031$.
\end{solution}
\begin{exercise}
Si consideri l'eq~$x^5+x=10$ (*).
\begin{enumerate}[(a)]
\item Mostrare che $\exists\, \xbar$ soluzione di (*) e che
tale $\xbar$ è unica.
\item Mostrare che $\xbar \in (0, 2)$.
\item Trovare $\xbar$ con errore minore a $10^{-2}$.
\end{enumerate}
\end{exercise}
\begin{solution}
Si consideri la funzione $f(x) = x^5 + x - 10$. Si osserva che
tale funzione è sempre continua. Si osserva che $f(0) = -10$ e che
$f(2) = 24$. Quindi $f$ ammette una soluzione $\xbar$ in
$(0, 2)$. \\
Si studia la derivata di $f$, ossia $f'(x) = 5x^4+1$. Poiché
$f'(x) > 0$ $\forall x \in \RR$, $f$ è strettamente crescente,
e quindi $f$ ammette un'unica soluzione, $\xbar$. \\
Per trovare la soluzione $\xbar$ con errore minore a $10^{-2}$,
come nell'esercizio precedente, è necessario applicare
il metodo di bisezione per $7$ volte, ricavando
$\xbar = 1.5469$.
\end{solution}
\begin{remark}
La scelta del punto medio nell'algoritmo di bisezione è (quasi)
forza. Nella costruzione degli intervalli è infatti
necessario che l'intervallo, all'infinito, tenda ad un solo
punto; qualora non venga scelto il punto medio degli intervalli,
questo non è assolutamente garantito.
\end{remark}
\begin{exercise}
Sia $I = [a, b]$. Siano $f_1$, $f_2 : I \to \RR$ continue tali che
$f_1(a) < f_2(a)$ e che $f_1(b) > f_2(b)$. Dimostrare che $\exists \xbar \in I$ tale che $f_1(\xbar) = f_2(\xbar)$.
\end{exercise}
\begin{solution}
Si consideri $g(x) = f_1(x) - f_2(x)$. $g$ è continua in $I$, e
$g(a) < 0$ e $g(b) > 0$ per ipotesi. Allora, per il teorema degli
zeri, $\exists x \in (a, b)$ tale che $g(x) = 0$, ossia
che $f_1(x) = f_2(x)$, da cui la tesi.
\end{solution}
\begin{exercise}
Sia $I = [a, b]$ e sia $f: I \to \RR$ continua. Sia $P$ un punto che si muove in modo continuo nella striscia $I \times \RR$. Sia in
particolare $P : [0, 1] \to I \times \RR$ tale che
$t \mapsto (x(t), y(t))$ con $a \leq x(t) \leq b$ $\forall t \in [0, 1]$, con $y(0) > f(a)$, $y(1) < f(b)$, $x(0) = a$ e $x(1) = b$. Dimostrare che $\exists t \in
[0, 1]$ tale che $(x(t), y(t)) = (x(t), f(x(t)))$, ossia che tale
curva si interseca con la funzione $f$.
\end{exercise}
\begin{solution}
Si consideri la funzione $g(t) = f(x(t)) - y(t)$. Poiché $x$ ed
$f$ sono continue, lo è anche la loro composizione, e così,
poiché anche $y$ è continua, lo è in particolare $g$. Dal momento
che $g(0) = f(x(0)) - y(0) = f(a) - y(0) < 0$ e $g(1) =
f(x(1)) - y(1) = f(b) - y(1) > 0$, per il teorema dei valori
intermedi, esiste $\tbar \in (0, 1)$ tale che $g(0) = 0$,
ossia tale che $f(x(\tbar)) = y(\tbar)$, da cui la tesi.
\end{solution}
\begin{exercise}
Sia $I = (a, b)$ e sia $f : (a, b) \to \RRbar$ continua tale che
$\exists \ell_a = \lim_{x \to a} f(x)$, $\ell_b = \lim_{x \to b} f(x)$.
Si consideri allora l'estensione continua $\tilde f$:
\[ \tilde f = \system{ f(x) & \text{se } x \neq a, b, \\ \ell_a & \text{se } x = a, \\ \ell_b & \text{se } x = b.} \]
Allora vale che $\tilde f$ è continua in $\overline I$\footnote{Come
già riscontrato, vale un risultato ancora più forte:
data un'estensione $\tilde f$ di $f$ in $\overline I$, $\tilde f$
è continua se e solo se i valori estesi sono esattamente i limiti
della funzione nei punti di $I \setminus \overline I$; e quindi
l'estensione continua è ben definita, e unica del suo genere.}.
\end{exercise}
\begin{solution}
Sicuramente $\tilde f$ è continua in $I$, dacché vale quanto
$f$ in questa porzione di intervallo. Poiché $\ell_a = \lim_{x \to a} f(x)$, per ogni intorno $I$ di $\ell_a$ esiste un intorno $J$ di $a$
tale che $f(J \cap I \setminus \{a\}) = f(J \cap I) = \tilde f(J \cap I) \subseteq I$, ossia, per definizione, $\tilde f$ è continua
anche in $a$, e, analogamente, anche in $b$.
\end{solution}
\begin{remark} Come mostrato nella traccia dell'esercizio precedente,
si possono estendere continuamente alcune funzioni elementari.
Per esempio, detta $f(x) = \frac{1}{x^2}$, si può estendere $f$
a $\tilde f : \RRbar \to \RRbar$ in modo tale che:
\[ \tilde f(x) = \system {0 & \text{se } x = \pm \infty, \\ +\infty & \text{se } x = 0, \\ f(x) & \text{altrimenti.}} \]
\end{remark}
%TODO: dimostrare che se limite sinistro e destro coincidono, allora esiste il limite ed è lo stesso del limite sinistro e destro.
\begin{exercise}
Si trovi un esempio di $f : X \to \RRbar$, dove, dato $\xbar$ punto
di accumulazione di $X$, $f(x) \tendsto{\xbar} \ell$, ma
$\exists (x_n) \subseteq X$ tale che $x_n \tendston \xbar$, ma
$f(x_n)$ non tende a $\ell$ per $n \to \infty$.
\end{exercise}
\begin{solution}
\end{solution}
\begin{exercise}
Sia $X \subseteq \RRbar$ tale che ogni punto di $X$ sia isolato.
Dimostrare allora che $X$ è al più numerabile.
\end{exercise}
\begin{solution}
Sia $\xbar \in X$. Poiché $\xbar$ è per ipotesi isolato, esiste
un intorno $I(\xbar)$ di $\xbar$ tale che $I \cap X = \{\xbar\}$. Si può
sempre trovare un intorno $J(\xbar)$ più piccolo di $I(\xbar)$ tale
che $J(\xbar) \cap I(x) = \emptyset$ $\forall x \in X \setminus \{\xbar\}$. Se infatti non si potesse, esisterebbe un $x \in X \setminus \{\xbar\}$ tale che $J \cap I(x) \neq \emptyset$ per ogni
intorno $J \subseteq I(\xbar)$ di $\xbar$: sicuramente tale $x \notin J$, altrimenti $I(\xbar)$ conterrebbe un elemento di $X$ diverso
da $\xbar$, assurdo dal momento che $I(\xbar)$ non ne contiene uno
per costruzione; ma $x$ non può neanche appartenere a $X \setminus J$,
dacché in tal modo si può sempre costruire con errore a piacimento
un intorno più piccolo di $I(x)$ tale che sia disgiunto con $J$,
\Lightning. Dal momento che $\QQ$ è denso in $\RRbar$, si può
sempre associare a $J(\xbar)$ un numero razionale $q$ al suo interno.
In questo modo si può costruire una funzione $f : X \to \QQ$,
tale che $f(\xbar) = q$. Poiché i $J(x)$ sono digiunti per costruzione,
$f$ è iniettiva, e quindi $\abs X \leq \abs \QQ = \abs \NN$, e quindi
$X$ è al più numerabile. %TODO: approfondire
\end{solution}
\end{document}

Binary file not shown.

@ -4,7 +4,7 @@
\title{\textbf{Note del corso di Fisica 1}}
\author{Gabriel Antonio Videtta}
\date{\today}
\date{29 e 30 marzo 2023}
\begin{document}
@ -42,4 +42,8 @@
$\omega$ sul punto applicato $A$ con vettore $\vec r$ il
vettore perpendicolare ad ambo i vettori $\vec r \times \vec v$.
\end{definition}
Si consideri $\vec{\ell_\omega} = (\vec r - \vec{r_0}) \times \vec p$.
Allora, la sua derivata è $(\vec v . \vec{r_0}) \times \vec p +
(\vec r - \vec{r_0}) \times \vec F$.
\end{document}

@ -74,6 +74,7 @@
\newcommand{\xbar}{\overline{x}}
\newcommand{\ybar}{\overline{y}}
\newcommand{\tbar}{\overline{t}}
\newcommand{\zbar}{\overline{z}}
\newcommand{\RRbar}{\overline{\RR}}

Loading…
Cancel
Save